5
$\begingroup$

Let $s \geq 0$ be fixed. The $J$-homomorphism includes $\pi_{8s+1}(SO) = \mathbb Z/2$ in $\pi_{8s+1}^s$, the $(8s+1)$-th stable homotopy group of spheres.

Now regard $\pi_{8s+1}^s = \pi_{8s+1} ((B\Sigma_{\infty})^+)$ , where $\Sigma_{\infty}$ is the group of compactly supported permutations of $\mathbb N$.

The obvious map (i.e. write permutations as permutation matrices) $\Sigma_{\infty} \to O$ induces a map $\pi_{8s+1}^s \to \pi_{8s+1}(BO) = \pi_{8s}(O)= \mathbb Z/2$.

Is the composition $\mathbb Z/2 = \pi_{8s+1} (SO) \to \pi_{8s+1}^s \to \pi_{8s} (O) = \mathbb Z/2$ trivial or nontrivial?

For $s = 0$ this map is nontrivial, since the two elements of $\pi_1^s$ correspond to odd and even permutations in $\Sigma_{\infty}$, whereas the two elements in $\pi_1(BO) = \pi_0(O)$ correspond to determinant $\pm1$ matrices and the map $\Sigma_{\infty} \to O$ preserves this structure.

So what happens for positive $s$? [I have a strong feeling that the map is then trivial, but maybe I am wrong...]

Remark: This map factors through $K_{8s+1}(\mathbb Z)$.

$\endgroup$

1 Answer 1

6
$\begingroup$

The composition is trivial. Composition with $\eta$ acts trivially on the source and nontrivially on the target, for positive $s$, which implies the claim. This argument does not apply for $s=0$, because $J : SO \to QS^0$ does not deloop.

$\endgroup$
2
  • $\begingroup$ Today, I was reading parts of Akhil Mathew's notes of a course Michael Hopkins taught in 2012, see math.berkeley.edu/~amathew/256y.pdf. On page 126, there is an exercise asking basically the same question as I did and the answer is the opposite of what you claimed. (There is no proof.) So what is the right answer, at the end of the day?? $\endgroup$ Jun 25, 2015 at 1:11
  • $\begingroup$ Proposition 7.19 on page 46 of J. Frank Adams' paper "On the groups J(X). IV", Topology 5 1966 21–71, states that the composite in question ($d_R J : \pi_r(SO) \to Z/2$ for $r \equiv 1 \mod 8$) is an isomorphism for $r=1$ and zero for $r>1$. I trust Adams in this (especially in view of the simple argument given above). $\endgroup$ Sep 11, 2015 at 14:17

Your Answer

By clicking “Post Your Answer”, you agree to our terms of service and acknowledge you have read our privacy policy.

Not the answer you're looking for? Browse other questions tagged or ask your own question.